Difference between revisions of "2015 AMC 10B Problems/Problem 14"

(Solution 3)
(Solution 3)
Line 17: Line 17:
 
*no math here*
 
*no math here*
 
There are 2 cases. Case 1 is that <math>(x-a)(x-b)=0</math> and <math>(x-b)(x-c)=0</math>. Lets test that 1st. If <math>x-b=0</math>, the maximum value for <math>x</math> and <math>b</math> is 9. Then b=9 and x=9. The next highest values are 7 and 8 so a=8 and c=9. Therefore, <math>\frac{18+8+7}{2}= \boxed{\textbf{(D)}~16.5}</math>.
 
There are 2 cases. Case 1 is that <math>(x-a)(x-b)=0</math> and <math>(x-b)(x-c)=0</math>. Lets test that 1st. If <math>x-b=0</math>, the maximum value for <math>x</math> and <math>b</math> is 9. Then b=9 and x=9. The next highest values are 7 and 8 so a=8 and c=9. Therefore, <math>\frac{18+8+7}{2}= \boxed{\textbf{(D)}~16.5}</math>.
By: Fas123456
 
  
 
==See Also==
 
==See Also==
 
{{AMC10 box|year=2015|ab=B|num-b=13|num-a=15}}
 
{{AMC10 box|year=2015|ab=B|num-b=13|num-a=15}}
 
{{MAA Notice}}
 
{{MAA Notice}}

Revision as of 17:17, 28 January 2018

Problem

Let $a$, $b$, and $c$ be three distinct one-digit numbers. What is the maximum value of the sum of the roots of the equation $(x-a)(x-b)+(x-b)(x-c)=0$?

$\textbf{(A) }15\qquad \textbf{(B) }15.5\qquad \textbf{(C) }16\qquad \textbf{(D) }16.5\qquad \textbf{(E) }17$

Solution

Solution 1

Expanding the equation and combining like terms results in $2x^2-(a+2b+c)x+(ab+bc)=0$. By Vieta's formula the sum of the roots is $\dfrac{-[-(a+2b+c)]}{2}=\dfrac{a+2b+c}{2}$. To maximize this expression we want $b$ to be the largest, and from there we can assign the next highest values to $a$ and $c$. So let $b=9$, $a=8$, and $c=7$. Then the answer is $\dfrac{8+18+7}{2}=\boxed{\textbf{(D)} 16.5}$.

Solution 2

Factoring out $(x-b)$ from the equation yields $(x-b)(2x-(a+c))=0 \Rightarrow (x-b)(x-\frac{a+c}{2})=0$. Therefore the roots are $b$ and $\frac{a+c}{2}$. Because $b$ must be the larger root to maximize the sum of the roots, letting $a,b,$ and $c$ be $8,9,$ and $7$ respectively yields the sum $9+\frac{8+7}{2} = 9+7.5 = \boxed{\textbf{(D)}~16.5}$.

Solution 3

  • no math here*

There are 2 cases. Case 1 is that $(x-a)(x-b)=0$ and $(x-b)(x-c)=0$. Lets test that 1st. If $x-b=0$, the maximum value for $x$ and $b$ is 9. Then b=9 and x=9. The next highest values are 7 and 8 so a=8 and c=9. Therefore, $\frac{18+8+7}{2}= \boxed{\textbf{(D)}~16.5}$.

See Also

2015 AMC 10B (ProblemsAnswer KeyResources)
Preceded by
Problem 13
Followed by
Problem 15
1 2 3 4 5 6 7 8 9 10 11 12 13 14 15 16 17 18 19 20 21 22 23 24 25
All AMC 10 Problems and Solutions

The problems on this page are copyrighted by the Mathematical Association of America's American Mathematics Competitions. AMC logo.png